(Completed solution)
(Corrected error regarding functional form of far inductors)
 
Line 38: Line 38:
 
\end{align}</math>
 
\end{align}</math>
  
Because the inductors are identical to each other, only three unique, '''constant''' entries exist in the inductance matrix <math>\mathbf{L}_{ss}</math>. The self-inductance <math>L_s</math> combines the effects of leakage inductance and magnetizing inductance. The mutual inductance between inductors ''near'' each other, <math>\frac{2\pi}{5} \, \text{rad}</math> apart in other words, is denoted <math>M_{ns}</math>. The mutual inductance between inductors ''far'' from each other, <math>\frac{4\pi}{5} \, \text{rad}</math> apart in other words, is denoted <math>M_{fs}</math>.
+
Because the inductors are identical to each other, only three unique, '''constant''' entries exist in the inductance matrix <math>\mathbf{L}_{ss}</math>. The self-inductance <math>L_s > 0</math> combines the effects of leakage inductance and magnetizing inductance. The mutual inductance between inductors ''near'' each other, <math>\frac{2\pi}{5} \, \text{rad}</math> apart in other words, is denoted <math>M_{ns}</math>. The mutual inductance between inductors ''far'' from each other, <math>\frac{4\pi}{5} \, \text{rad}</math> apart in other words, is denoted <math>M_{fs}</math>. Because inductors with spacing <math> -\frac{\pi}{2} < \Delta\theta < +\frac{\pi}{2}</math> produce a positive value of cosine, the mutual inductance of near conductors is positive <math>M_{ns} > 0</math>. The inductors with spacing <math>-\pi < \Delta\theta < -\frac{\pi}{2}</math> or <math>+\frac{\pi}{2} < \Delta\theta < \pi</math> produce a negative value of cosine, leading to a negative mutual inductance <math>M_{fs} < 0</math>.
  
 
===<small>Solving Imbalanced Conditions</small>===
 
===<small>Solving Imbalanced Conditions</small>===
Line 75: Line 75:
 
v_1(t) &= L_s \frac{d}{dt} \left[10 \cos\left(\omega_1 t\right) \, \text{A}\right] \\
 
v_1(t) &= L_s \frac{d}{dt} \left[10 \cos\left(\omega_1 t\right) \, \text{A}\right] \\
 
v_1(t) &= L_s \left[-10 \omega_1 \sin\left(\omega_1 t\right) \, \text{A}\right] \\
 
v_1(t) &= L_s \left[-10 \omega_1 \sin\left(\omega_1 t\right) \, \text{A}\right] \\
v_1(t) &= 10^3 L_s \cos\left(100 \, \frac{\text{rad}}{\text{s}} t - \frac{\pi}{2} \, \text{rad}\right) \, \text{V}
+
v_1(t) &= 10^3 L_s \cos\left(100 \, \frac{\text{rad}}{\text{s}} t - \frac{\pi}{2} \, \text{rad}\right) \, \text{V} \\
 +
v_2(t) &= 10^3 M_{ns} \cos\left(100 \, \frac{\text{rad}}{\text{s}} t - \frac{\pi}{2} \, \text{rad}\right) \, \text{V} \\
 +
v_4(t) &= 10^3 M_{fs} \cos\left(100 \, \frac{\text{rad}}{\text{s}} t - \frac{\pi}{2} \, \text{rad}\right) \, \text{V} \\
 +
v_4(t) &= 10^3 |M_{fs}| \cos\left(100 \, \frac{\text{rad}}{\text{s}} t + \frac{\pi}{2} \, \text{rad}\right) \, \text{V}
 
\end{align}</math>
 
\end{align}</math>
  
Similar calculations unfold for <math>v_2(t)</math> and <math>v_4(t)</math>, just with different amplitudes. The voltages all have a phase shift <math>\phi_j = -\frac{\pi}{2} \, \text{rad} \, \forall j \in \{1, 2, 3, 4, 5\}</math>. In other words, the current lags the voltage with quadrature phase shift.
+
Similar calculations unfold for <math>v_2(t)</math> and <math>v_4(t)</math>, just with different amplitudes. The voltages all have a phase shift <math>\phi_j = -\frac{\pi}{2} \, \text{rad} \, j \in \{1, 2, 5\}</math> and <math>\phi_j = +\frac{\pi}{2} \, \text{rad} \, j \in \{3, 4\}</math>. In other words, the current lags the voltage with quadrature phase shift for inductors shifted <math> -\frac{\pi}{2} < \Delta\theta < +\frac{\pi}{2}</math>, and the current leads the voltage with quadrature phase shift for the remaining inductors.
  
 
<math>\begin{equation}
 
<math>\begin{equation}
Line 89: Line 92:
  
 
<math>\begin{equation}
 
<math>\begin{equation}
\boxed{\phi_3 = -\frac{\pi}{2} \, \text{rad}}
+
\boxed{\phi_4 = +\frac{\pi}{2} \, \text{rad}}
 
\end{equation}</math>
 
\end{equation}</math>
  

Latest revision as of 15:22, 4 August 2018


Answers and Discussions for

ECE Ph.D. Qualifying Exam PE-1 August 2014



Problem 1, part a

Voltage Calculation

The voltage equation for these five identical, magnetically coupled inductors may be written down in matrix form (resistance is neglected).

$ \begin{align} \vec{v}_{12345s} &= \cancelto{\mathbf{0}}{\mathbf{r}_s} \vec{i}_{12345s} + p \vec{\lambda}_{12345s} \\ \vec{v}_{12345s} &= p \vec{\lambda}_{12345s} \end{align} $

The flux linkage equation needs to be written (linear magnetics are assumed).

$ \begin{align} \vec{\lambda}_{12345s} &= \mathbf{L}_{ss} \vec{i}_{12345s} \\ \begin{bmatrix} \lambda_1 \\ \lambda_2 \\ \lambda_3 \\ \lambda_4 \\ \lambda_5 \end{bmatrix} &= \begin{bmatrix} L_s & M_{ns} & M_{fs} & M_{fs} & M_{ns} \\ M_{ns} & L_s & M_{ns} & M_{fs} & M_{fs} \\ M_{fs} & M_{ns} & L_s & M_{ns} & M_{fs} \\ M_{fs} & M_{fs} & M_{ns} & L_s & M_{ns} \\ M_{ns} & M_{fs} & M_{fs} & M_{ns} & L_s \end{bmatrix} \begin{bmatrix} i_1 \\ i_2 \\ i_3 \\ i_4 \\ i_5 \end{bmatrix} \end{align} $

Because the inductors are identical to each other, only three unique, constant entries exist in the inductance matrix $ \mathbf{L}_{ss} $. The self-inductance $ L_s > 0 $ combines the effects of leakage inductance and magnetizing inductance. The mutual inductance between inductors near each other, $ \frac{2\pi}{5} \, \text{rad} $ apart in other words, is denoted $ M_{ns} $. The mutual inductance between inductors far from each other, $ \frac{4\pi}{5} \, \text{rad} $ apart in other words, is denoted $ M_{fs} $. Because inductors with spacing $ -\frac{\pi}{2} < \Delta\theta < +\frac{\pi}{2} $ produce a positive value of cosine, the mutual inductance of near conductors is positive $ M_{ns} > 0 $. The inductors with spacing $ -\pi < \Delta\theta < -\frac{\pi}{2} $ or $ +\frac{\pi}{2} < \Delta\theta < \pi $ produce a negative value of cosine, leading to a negative mutual inductance $ M_{fs} < 0 $.

Solving Imbalanced Conditions

The current through the circuit is given knowing that four the the five inductors are open-circuited and carry no current.

$ \begin{equation} \begin{bmatrix} i_1(t) \\ i_2(t) \\ i_3(t) \\ i_4(t) \\ i_5(t) \end{bmatrix} = \begin{bmatrix} 10 \cos\left(100 \, \frac{\text{rad}}{\text{s}} t\right) \\ 0 \\ 0 \\ 0 \\ 0 \end{bmatrix} \, \text{A} \end{equation} $

The following, partially complete information is measured from the circuit.

$ \begin{equation} \begin{bmatrix} v_1(t) \\ v_2(t) \\ v_3(t) \\ v_4(t) \\ v_5(t) \end{bmatrix} = \begin{bmatrix} 40 \cos(\omega_1 t + \phi_1) \\ 10 \cos(\omega_2 t + \phi_2) \\ v_3(t) \\ 1 \cos(\omega_4 t + \phi_4) \\ v_5(t) \end{bmatrix} \, \text{V} \end{equation} $

Because of the linear magnetics, the frequency of the voltage response must match the frequency of the current excitation: $ \omega_j = 100 \, \frac{\text{rad}}{\text{s}} \, \forall j \in \{1, 2, 3, 4, 5\} $.

$ \begin{equation} \boxed{\omega_1 = 100 \, \frac{\text{rad}}{\text{s}}} \end{equation} $

$ \begin{equation} \boxed{\omega_2 = 100 \, \frac{\text{rad}}{\text{s}}} \end{equation} $

$ \begin{equation} \boxed{\omega_4 = 100 \, \frac{\text{rad}}{\text{s}}} \end{equation} $

By analyzing the voltage equation and flux linkage equation, the phase shift of the voltage of each coil becomes known. The first coil is analyzed.

$ \begin{align} v_1(t) &= p \lambda_1(t) \\ v_1(t) &= p \left[L_s i_1(t)\right] \\ v_1(t) &= L_s \frac{d}{dt} \left[10 \cos\left(\omega_1 t\right) \, \text{A}\right] \\ v_1(t) &= L_s \left[-10 \omega_1 \sin\left(\omega_1 t\right) \, \text{A}\right] \\ v_1(t) &= 10^3 L_s \cos\left(100 \, \frac{\text{rad}}{\text{s}} t - \frac{\pi}{2} \, \text{rad}\right) \, \text{V} \\ v_2(t) &= 10^3 M_{ns} \cos\left(100 \, \frac{\text{rad}}{\text{s}} t - \frac{\pi}{2} \, \text{rad}\right) \, \text{V} \\ v_4(t) &= 10^3 M_{fs} \cos\left(100 \, \frac{\text{rad}}{\text{s}} t - \frac{\pi}{2} \, \text{rad}\right) \, \text{V} \\ v_4(t) &= 10^3 |M_{fs}| \cos\left(100 \, \frac{\text{rad}}{\text{s}} t + \frac{\pi}{2} \, \text{rad}\right) \, \text{V} \end{align} $

Similar calculations unfold for $ v_2(t) $ and $ v_4(t) $, just with different amplitudes. The voltages all have a phase shift $ \phi_j = -\frac{\pi}{2} \, \text{rad} \, j \in \{1, 2, 5\} $ and $ \phi_j = +\frac{\pi}{2} \, \text{rad} \, j \in \{3, 4\} $. In other words, the current lags the voltage with quadrature phase shift for inductors shifted $ -\frac{\pi}{2} < \Delta\theta < +\frac{\pi}{2} $, and the current leads the voltage with quadrature phase shift for the remaining inductors.

$ \begin{equation} \boxed{\phi_1 = -\frac{\pi}{2} \, \text{rad}} \end{equation} $

$ \begin{equation} \boxed{\phi_2 = -\frac{\pi}{2} \, \text{rad}} \end{equation} $

$ \begin{equation} \boxed{\phi_4 = +\frac{\pi}{2} \, \text{rad}} \end{equation} $


Discussion



Back to PE-1, August 2014

Alumni Liaison

Ph.D. on Applied Mathematics in Aug 2007. Involved on applications of image super-resolution to electron microscopy

Francisco Blanco-Silva